1 Introduction

In this paper, we are interested in the multiplicity of solutions to the following nonhomogeneous p-Kirchhoff elliptic problem:

$$ \biggl(a+\lambda \biggl( \int_{\mathbb{R}^{N}} \bigl(\vert \nabla{u} \vert ^{p}+\vert u \vert ^{p} \bigr)\,dx \biggr)^{m} \biggr) \bigl(- \Delta_{p}u+\vert u\vert ^{p-2}u \bigr) =f(u)+h(x),\quad x\in \mathbb{R}^{N}, $$
(1.1)

where \(\Delta_{p}u=\operatorname {div}(\vert \nabla{u} \vert ^{p-2}\nabla{u})\) is the p-Laplacian operator, and the nontrivial function \(h(x)\) can be seen as a perturbation term. Problem (1.1) is a generalization of the model introduced by Kirchhoff [2]. More precisely, Kirchhoff proposed the model given by the equation

$$ \rho_{tt}- \biggl(\frac{P_{0}}{h}+\frac{E}{2L} \int^{L}_{0}u^{2}_{x}\,dx \biggr)u_{xx}=0,\quad 0< x< L, t>0, $$
(1.2)

which takes into account the changes in length of string produced by transverse vibration. The parameters in (1.2) have the following meaning: L is the length of the string, h is the area of cross-section, E is the Young modulus of material, ρ is the mass density, and \(P_{0}\) is the initial tension.

The equation

$$ \rho_{tt}-M \bigl(\Vert \nabla u\Vert ^{2}_{2} \bigr)\Delta u=f(x,u), \quad x\in\Omega, t>0, $$
(1.3)

generalizes equation (1.2), where \(M: \mathbb{R}^{+}\to\mathbb{R}\) is a given function, Ω is a domain of \(\mathbb{R}^{N}\). The stationary counterpart of (1.3) is the Kirchhoff-type elliptic equation

$$ -M \bigl(\Vert \nabla u\Vert ^{2}_{2} \bigr) \Delta u=f(x,u), \quad x\in\Omega, t>0. $$
(1.4)

Some classical and interesting results on Kirchhoff-type elliptic equations can be found, for example, in [39].

Particularly, Li et al. [10] considered the Kirchhoff-type problem

$$ \biggl(a+\lambda \biggl( \int_{\mathbb{R}^{N}} \bigl(\vert \nabla{u} \vert ^{2}+b \vert u\vert ^{2} \bigr)\,dx \biggr) \biggr) (-\Delta u+bu) =f(u),\quad x \in \mathbb{R}^{N}, $$
(1.5)

where \(N \geq3\), with constants \(a, b > 0\) and \(\lambda\geq0\) under the following assumptions:

\((H_{1})\) :

\(f\in C(\mathbb{R}^{+},\mathbb{R}^{+})\), \(\vert f(t)\vert \leq C(1+t^{q-1})\) for all \(t\in\mathbb{R}^{+}=[0,+\infty)\) and some \(q\in(2,2^{*})\), where \(2^{*}=\frac{2N}{N-2}\) for \(N\geq3\);

\((H_{2})\) :

\(\lim_{t\rightarrow0}\frac{f(t)}{t}=0\); \(\lim_{t\rightarrow+\infty}\frac{f(t)}{t}=+\infty\).

It is easy to see that \(f(u)=\vert u\vert ^{q-2}u\), \(2< q<4\), and \(N=3\) satisfy these conditions. They obtained that there exists \(\lambda _{0} > 0\) such that, for any \(\lambda\in[0, \lambda_{0})\), problem (1.5) has at least one positive solution in \(W^{1,2}({\mathbb {R}}^{N})\). The \(\lambda_{0}\) depends on f, a, b, the Sobolev constant, and several test functions in [10]; it is not very clear whether the the existence of solutions for (1.5) still holds for large \(\lambda >0\). Recently, Chen et al. [11] studied the existence of positive solutions to the p-Kirchhoff problem

$$ \textstyle\begin{cases} (a+\lambda( \int_{\mathbb{R}^{N}} (\vert \nabla u\vert ^{p}+b \vert u\vert ^{p} )\,dx )^{\tau} ) (- \Delta_{p}u+b\vert u\vert ^{p-2}u ) \\ \quad =\vert u\vert ^{m-2}u+\mu \vert u\vert ^{q-2}u, \quad x\in \mathbb{R}^{N}, \\ u(x)>0, \quad x\in\mathbb{R}^{N},\quad \quad u(x)\in W^{1,p} ( \mathbb{R}^{N} ), \end{cases} $$
(1.6)

where \(a,b>0\), \(\tau,\lambda\ge0\), \(\mu\in\mathbb{R}\), and \(1< p< N\). By the Nehari manifold method, they proved that problem (1.6) admits at least a positive ground state solution for any \(\lambda>0\) when \(p(\tau +1)< q< m< p^{*}=\frac{pN}{N-p}\). However, does the existence of solutions for (1.5) still hold for any \(\lambda>0\) when \(p< q< p(\tau+1)\) and \(\mu=0\)? This is a interesting problem. In this paper, we answer positively this question. More interesting results for Kirchhoff-type problems can be found in [1, 2, 57, 1014].

In the present paper, we are ready to extend the analysis to the nonhomogeneous p-Kirchhoff-type equation of (1.1) in \(\mathbb{R}^{N}\) with the nonlinearity \(f(u)\) satisfying the following conditions:

\((F_{1})\) :

\(f\in C(\mathbb{R}^{+},\mathbb{R}^{+})\), \(\vert f(t)\vert \leq C(t^{p-1}+t^{q-1})\) for all \(t\in\mathbb{R}^{+}\) and some \(q\in(p,p^{*})\), where \(p^{*}={pN}/({N-p})\), \(1< p< N\);

\((F_{2})\) :

\(\lim_{t\rightarrow0^{+}}\frac{f(t)}{t^{p-1}}=0\);

\((F_{3})\) :

\(\lim_{t\rightarrow+\infty}\frac{f(t)}{t^{p-1}}=+\infty\).

In addition, we suppose that the nontrivial and nonnegative function \(h(x)\equiv h(\vert x\vert )\in C^{1}(\mathbb{R}^{N})\cap L^{p'}(\mathbb{R}^{N})\) satisfies

\((H)\) :

there exists \(\xi(x)\in L^{p'}(\mathbb{R}^{N})\cap W^{1,\infty}(\mathbb{R}^{N})\) such that

$$ \bigl\vert \nabla h(x)\cdot x \bigr\vert \leq\xi^{p'}(x), \quad \forall x \in\mathbb{R}^{N}, $$
(1.7)

with \(p'=\frac{p}{p-1}\).

We will use the Ekeland variational principle [15] and a version of the mountain pass theorem in [1] to study the existence of multiple solutions of problem (1.1) in \({\mathbb {R}}^{N}\). It is well known that an important technical condition to get a bounded (PS) sequence is the following Ambrosetti-Rabinowitz-type condition (AR): there exists \(\theta>p\) such that \(0<\theta F(s)\le sf(s)\) for \(s>0\). The loss of (AR) condition renders variational techniques more delicate. Inspired by [1, 10], we use a cut-off functional and obtain a bounded (PS) sequence.

In order to state our main result, we introduce some Sobolev spaces and norms. Let \(W^{1,p}(\mathbb{R}^{N})\) be the usual Sobolev space with the norm

$$ \Vert u\Vert = \biggl( \int_{\mathbb{R}^{N}}\vert \nabla u\vert ^{p}+\vert u \vert ^{p}\,dx \biggr)^{\frac{1}{p}}, \quad 1< p< \infty. $$
(1.8)

We denote by \(\Vert \cdot \Vert _{q}\) the usual \(L^{q}({\mathbb {R}}^{N})\) norm. Then it well known that the embedding \(W^{1,p}(\mathbb {R}^{N})\hookrightarrow L^{q}(\mathbb{R}^{N})\) is continuous for \(q\in (p,p^{*}]\) and there exists a constant \(S_{q}\) such that

$$ \Vert u\Vert _{q}\leq S_{q}\Vert u \Vert , \quad \forall u\in W^{1,p} \bigl(\mathbb{R}^{N} \bigr). $$
(1.9)

Let \(X=W_{r}^{1,p}(\mathbb{R}^{N})\) be the subspace of \(W^{1,p}(\mathbb {R}^{N})\) containing only the radial functional. Then by the Lemma 2.2 in [11] we have that the embedding \(X\hookrightarrow L^{q}(\mathbb {R}^{N})\) is compact for \(q\in(p,p^{*})\).

A function \(u\in X\) is said to be a weak solution of (1.1) if for all \(v \in X\),

$$ \bigl(a+\lambda \Vert u\Vert ^{pm} \bigr) \int_{\mathbb{R}^{N}} \bigl(\vert \nabla u\vert ^{p-2}\nabla u \nabla v+\vert u\vert ^{p-2}uv \bigr) \,dx= \int_{\mathbb{R}^{N}} \bigl(f(u)+h \bigr)v \,dx. $$
(1.10)

Let \(I(u):X\rightarrow\mathbb{R} \) be the energy functional associated with problem (1.1) defined by

$$ I(u)=\frac{a}{p}\Vert u\Vert ^{p}+\frac{\lambda}{p(m+1)}\Vert u \Vert ^{p(m+1)}- \int_{\mathbb{R}^{N}} \bigl(F(u)+hu \bigr)\,dx, $$
(1.11)

where \(F(u)=\int_{0}^{u}f(s)\,ds\). It is easy to see that the functional \(I\in C^{1}(X,\mathbb{R})\) and its Gateaux derivative is given by

$$ \begin{aligned}[b] I'(u)v&= \bigl(a+\lambda \Vert u\Vert ^{pm} \bigr) \int_{\mathbb{R}^{N}} \bigl(\vert \nabla u\vert ^{p-2} \nabla u \nabla v+\vert u\vert ^{p-2}uv \bigr)\,dx \\ &\quad{} - \int_{\mathbb{R}^{N}} \bigl(f(u)+h \bigr)v\,dx, \quad \forall v\in X. \end{aligned} $$
(1.12)

Clearly, we see that a weak solution of (1.1) corresponds to a critical point of the functional.

The main result in this paper is as follows.

Theorem 1.1

Let \((F_{1})\)-\((F_{3})\) and \((H)\) hold. Then, there exist \(\lambda_{0}, \widetilde{{m}}_{0}>0\) such that, for any \(\lambda\in[0,\lambda_{0})\), (1.1) has at least two nontrivial solutions in X when \(\Vert h\Vert _{p'}<\widetilde{{m}}_{0}\).

Furthermore, consider \(h(x)=0\) and \(f(x,u)=\vert u\vert ^{q-2}u\), \(p< q<\min\{p(m+1),p^{*}\}\), that is,

$$ \biggl(a+\lambda \biggl( \int_{\mathbb{R}^{N}} \bigl(\vert \nabla{u} \vert ^{p}+\vert u \vert ^{p} \bigr)\,dx \biggr)^{m} \biggr) \bigl(- \Delta_{p}u+\vert u\vert ^{p-2}u \bigr) =\vert u\vert ^{q-2}u, \quad x\in\mathbb{R}^{N}. $$
(1.13)

We can now state the second main result.

Theorem 1.2

Let \(a>0\) and \(p< q<\min\{p(m+1),p^{*}\}\). Then there exists \(\lambda ^{*}>0\) such that problem (1.13) has at least one nontrivial solution for any \(\lambda\in(0, \lambda^{*}]\) and has no nontrivial weak solutions for any \(\lambda\in(\lambda^{*}, +\infty)\).

Remark 1.3

In [11], Chen and Zhu considered the case \(p< p(m+1)< q<p^{*}\). They proved that problem (1.1) admits at least one positive solution for any \(\lambda>0\).

2 Proof of Theorem 1.1

In this section, we first establish some properties of the functional I and then prove Theorem 1.1. Throughout the paper, we denote by C or \(C_{i}\) s positive constants that may vary from line to line and are not essential to the problem.

Lemma 2.1

If assumptions \((F_{1})\)-\((F_{3})\) hold and \(h(x)\in L^{p'}(\mathbb{R}^{N})\), then there exist \(\rho, \alpha, m_{0}>0\) such that \(I(u)\geq\alpha>0\) with \(\Vert u\Vert =\rho\) and \(\Vert h\Vert _{p'}< m_{0}\).

Proof

It follows from \((F_{1})\)-\((F_{2})\) that

$$ F(s)\leq\varepsilon \vert s\vert ^{p}+C_{\varepsilon} \vert s \vert ^{q}, \quad \forall s\in\mathbb{R}, $$
(2.1)

with \(\varepsilon>0\). By the Hölder inequality we have

$$ \biggl\vert \int_{\mathbb{R}^{N}}hu\,dx \biggr\vert \leq S_{q}^{-1} \Vert h\Vert _{p'}\Vert u\Vert \leq\epsilon \Vert u\Vert ^{p}+C_{\epsilon} \Vert h\Vert _{p'}^{p'}. $$
(2.2)

Thus,

$$ \begin{aligned}[b] I(u)&\geq\frac{a}{p}\Vert u\Vert ^{p}- \varepsilon \Vert u\Vert ^{p}-C_{\varepsilon} \Vert u\Vert ^{q}-\epsilon \Vert u\Vert ^{p}-C_{\epsilon} \Vert h \Vert _{p'}^{p'} \\ &\geq\frac{a}{2p}\Vert u\Vert ^{p}-C_{1}\Vert u\Vert ^{q}-C_{2} \Vert h\Vert ^{p'}_{p'}, \end{aligned} $$
(2.3)

where \(\varepsilon=\epsilon=\frac{a}{4p}\), \(C_{1}\), \(C_{2}\) are some positive constants. Let

$$ z(t)=\frac{a}{2p}t^{p}-C_{1}t^{q}, \quad t\geq0. $$
(2.4)

We see that there exists \(\rho>0\) such that \(\max_{t\geq0}z(t)=z(\rho)\equiv m_{0}>0\). Then it follows from (2.3) that there exists \(\alpha>0\) such that \(I(u)\geq\alpha\) with \(\Vert u\Vert =\rho\) and \(\Vert h\Vert _{p'}< m_{0}\). This ends the proof of Lemma 2.1. □

We denote by \(B_{r}\) the open ball in X centered at the origin with radius r. By Ekland’s variational principle [15] we get the following lemma, which implies that there exists a function \(u_{0}\) such that \(I'(u_{0})=0\) and \(I(u_{0})<0\) if \(\Vert h\Vert _{p'}\) is small.

Lemma 2.2

Let assumptions \((F_{1})\)-\((F_{3})\) hold, and \(h(x)\in L^{p'}(\mathbb{R}^{N})\), \(h(x)\not\equiv0\), with \(\Vert h\Vert _{p'}< m_{0}\). Then there exists a function \(u_{0}\in X\) such that

$$ I(u_{0})=\inf \bigl\{ I(u):u\in{\overline{B}_{\rho}} \bigr\} < 0, $$
(2.5)

and \(u_{0}\) is a nontrivial weak solution of problem (1.1).

Proof

Choose a function \(\phi\in C_{0}^{1}(\mathbb{R}^{N})\) such that \(\int_{\mathbb{R}^{N}}h(x)\phi(x)\,dx>0\). Then

$$ I(t\phi)\leq\frac{a}{p}t^{p}\Vert \phi \Vert ^{p}+ \frac{\lambda}{p(m+1)}t^{p(m+1)}\Vert \phi \Vert ^{p(m+1)}-t \int_{\mathbb{R}^{N}}h(x)\phi \,dx< 0 $$
(2.6)

for small \(t>0\) and thus for any open ball \({B}_{\kappa}\subset X\) such that \(-\infty< c_{\kappa}=\inf_{\overline{B}_{\kappa}}I(u)<0\). Thus,

$$ c_{\rho}=\inf_{u\in\overline{B}_{\rho}}I(u)< 0 \quad \mbox{and}\quad \inf _{u\in\partial B_{\rho}}I(u)>0, $$
(2.7)

where ρ is given in Lemma 2.1. Let \(\varepsilon_{n}\downarrow0\) be such that

$$ 0< \varepsilon_{n}< \inf_{u\in\partial B_{\rho}}I(u)- \inf_{u\in B_{\rho}}I(u). $$
(2.8)

Then, by Ekland’s variational principle [15] there exists \(\{u_{n}\} \subset\overline{B}_{\rho}\) such that

$$ c_{\rho}\leq I(u_{n})< c_{\rho}+ \varepsilon_{n} $$
(2.9)

and

$$ I(u_{n})< I(u)+\varepsilon_{n}\Vert u_{n}-u\Vert \qquad\mbox{for all } u\in\overline{B_{\rho}}, u_{n}\neq u. $$
(2.10)

Then, it follows from (2.8)-(2.10) that

$$ I(u_{n})< c_{\rho}+\varepsilon_{n}\leq\inf _{u\in B_{\rho}}I(u)+ \varepsilon_{n}< \inf_{u\in\partial B_{\rho}}I(u). $$
(2.11)

So \(u_{n}\in B_{\rho}\), and we now consider the function \(F: \overline {B}_{\rho}\to {\mathbb {R}}\) given by

$$ F(u)=I(u)+\varepsilon_{n}\Vert u_{n}-u\Vert , \quad u\in \overline{B}_{\rho}. $$
(2.12)

Then (2.10) shows that \(F(u_{n})< F(u)\), \(u\in\overline{{B}}_{\rho}\), \(u_{n}\neq u\), and thus \(u_{n}\) is a strict local minimum of F. Moreover,

$$ t^{-1} \bigl(F(u_{n}+tv)-F(u_{n}) \bigr)\geq0 \quad\mbox{for small } t>0, \forall v\in B_{1}. $$
(2.13)

Hence,

$$ t^{-1} \bigl(I(u_{n}+tv)-I(u_{n}) \bigr)+ \varepsilon_{n}\Vert v\Vert \geq0. $$
(2.14)

Passing to the limit as \(t\to0^{+}\), it follows that

$$ I'(u_{n})v+\varepsilon_{n} \Vert v\Vert \geq0,\quad\forall v\in B_{1}. $$
(2.15)

Replacing v in (2.15) by −v, we get

$$ -I'(u_{n})v+\varepsilon_{n}\Vert v\Vert \geq0,\quad\forall v\in B_{1}, $$
(2.16)

so that \(\Vert I'(u_{n})\Vert \leq\varepsilon_{n}\). Therefore, there is a sequence \(\{u_{n}\}\in B\rho\) such that \(I(u_{n})\to c_{\rho}<0\) and \(I'(u_{n})\to0\) in \(X^{*}\) as \(n\to\infty\). In the following, we will prove that \(\{u_{n}\}\) has a convergent subsequence in X. Indeed, since \(\Vert u_{n}\Vert <\rho\), by the reflexivity of X and compact embedding \(X\hookrightarrow L^{q}\) for all \(q\in(p,p^{*})\), passing to a subsequence, we can assume that

$$ u_{n}\rightharpoonup u_{0}, \quad \text{in } X; \quad\quad u_{n} \rightarrow u_{0},\quad L^{q} \bigl( \mathbb{R}^{N} \bigr); \quad\quad u_{n}\rightarrow u_{0}, \quad \mbox{a.e. in } \mathbb{R}^{N}. $$
(2.17)

By (1.12) we can get

$$ \bigl(I(u_{n})-I(u_{0}) \bigr)'(u_{n}-u_{0})=P_{n}+Q_{n}+K_{n}, $$
(2.18)

where

$$ \begin{gathered} P_{n} = \bigl(a+\lambda \Vert u_{n}\Vert ^{pm} \bigr) \int_{\mathbb{R}^{N}} \bigl(\vert \nabla u_{n}\vert ^{p-2}\nabla u_{n}-\vert \nabla u\vert ^{p-2} \nabla u \bigr)\nabla(u_{n}-u_{0}) \\ \hphantom{P_{n} =}{} + \bigl(\vert u_{n}\vert ^{p-2}u_{n}-u_{0}^{p-2}u_{0} \bigr) (u_{n}-u_{0})\,dx, \\ Q_{n}= \lambda \bigl( \bigl(\Vert u_{n}\Vert ^{pm}-\Vert u_{0}\Vert ^{pm} \bigr) \bigr) \int_{\mathbb{R}^{N}}\vert \nabla u_{0}\vert ^{p-2} \nabla{u_{0}}\nabla(u_{n}-u_{0}) \\ \hphantom{Q_{n}=}{}+\vert u_{0}\vert ^{p-2}u_{0}(u_{n}-u_{0})\,dx, \\ K_{n}= \int_{\mathbb{R}^{N}} \bigl(f(u_{n})-f(u_{0}) \bigr) (u_{n}-u_{0})\,dx. \end{gathered} $$
(2.19)

It is clear that

$$ \bigl(I(u_{n})-I(u_{0}) \bigr)'(u_{n}-u_{0}) \rightarrow0 \quad\text{as } n\rightarrow\infty. $$
(2.20)

By \((F_{1})\) and \((F_{2})\), for any \(\varepsilon>0\), there exists \(C_{\varepsilon}>0\) such that

$$ \bigl\vert f(t) \bigr\vert \leq\varepsilon \vert t\vert ^{p-1}+C_{\varepsilon} \vert t\vert ^{q-1},\quad t\in \mathbb{R}. $$
(2.21)

Hence,

$$\begin{aligned} \begin{aligned}[b] \vert K_{n}\vert &= \biggl\vert \int_{\mathbb{R}^{N}} \bigl(f(u_{n})-f(u_{0}) \bigr) (u_{n}-u_{0})\,dx \biggr\vert \\ &\leq\varepsilon \bigl(\Vert u_{n}\Vert ^{p-1}+\Vert u_{0}\Vert ^{p-1} \bigr)\Vert u_{n}-u_{0} \Vert +C_{\varepsilon}\bigl(\Vert u_{n}\Vert ^{q-1}_{q}+\Vert u_{0}\Vert ^{q-1}_{q} \bigr)\Vert u_{n}-u_{0} \Vert _{q} \\ &\to0 \quad \mbox{as } n\to\infty. \end{aligned} \end{aligned}$$
(2.22)

Define the linear function \(g:X\rightarrow\mathbb{R}\) by

$$ g(\omega)= \int_{\mathbb{R}^{N}}\vert \nabla u_{0}\vert ^{p-2} \nabla u_{0}\nabla\omega+\vert u_{0}\vert ^{p-2}u_{0}\omega \,dx. $$
(2.23)

Noticing that \(\vert g(\omega)\vert \leq2\Vert u_{0}\Vert ^{p-1}\Vert \omega \Vert \), we can deduce that g is continuous on X. Using \(u_{n}\rightharpoonup u_{0}\) in X, we have

$$ \begin{aligned}[b] g(u_{n}-u_{0})&= \int_{\mathbb{R}^{N}}\vert \nabla u_{0}\vert ^{p-2} \nabla u_{0}\nabla(u_{n}-u_{0})+\vert u_{0}\vert ^{p-2}u_{0}(u_{n}-u_{0})\,dx \\ & \rightarrow0 \quad \mbox{as } n\rightarrow\infty. \end{aligned} $$
(2.24)

Since \(\Vert u_{n}\Vert <\rho\), we deduce that \(\vert Q_{n}\vert \rightarrow0\) as \(n\rightarrow\infty\).

Combining the above results, we have \(\vert P_{n}\vert \to0\) as \(n\to\infty\), Then, using the standard inequalities in \(\mathbb{R}^{N}\)

$$\begin{aligned} \begin{aligned} &\bigl\langle \vert x\vert ^{p-2}x- \vert y\vert ^{p-2}y,x-y \bigr\rangle \geq{C_{p}}\vert x-y\vert ^{p}, \quad p\geq2, \\ &\bigl\langle \vert x\vert ^{p-2}x-\vert y\vert ^{p-2}y,x-y \bigr\rangle \geq\frac{C_{p}\vert x-y\vert ^{p}}{{\vert x\vert +\vert y\vert }^{2-p}},\quad 2>p>1, \end{aligned} \end{aligned}$$
(2.25)

where \(\langle\cdot,\cdot\rangle\) denotes the scalar product in \(\mathbb{R}^{N}\), we can show that \(u_{n} \rightarrow u_{0}\) in X. Thus, \(u_{0}\) is a nontrivial weak solution of problem (1.1). The proof is completed. □

Next, we prove that problem (1.1) has a mountain-pass-type solution. To overcome the difficulty of finding a bounded (PS) sequence for the associated functional I, motivated by [1, 10], we use a cut-off function \(\psi\in C_{0}^{1}(\mathbb{R}^{+})\) that satisfies

$$ \begin{aligned} &\psi(t)=1, \quad \forall t\in[0,1]; \quad\quad 0\leq\psi\leq1, \quad \forall t\in(1,2); \\& \psi(t)\equiv0,\quad \forall t \in[2,+\infty); \quad\quad \bigl\Vert \psi' \bigr\Vert _{\infty}\leq2, \end{aligned} $$
(2.26)

and study the following modified functional \(I^{T}\) defined by

$$ I^{T}(u)=\frac{a}{p}\Vert u\Vert ^{p}+ \frac{\lambda}{p(m+1)}\eta_{T}(u)\Vert u\Vert ^{p(m+1)}- \int_{\mathbb{R}^{N}} \bigl(F(u)+hu \bigr)\,dx, \quad u\in X, $$
(2.27)

where \(T>0\) and \(\eta_{T}(u)=\psi(\frac{\Vert u\Vert ^{p}}{T^{p}})\). For \(T>0\) sufficiently large and λ sufficiently small, we will prove that there exists a critical point \(\tilde{u}_{0}\) of \(I_{T}\) such that \(\Vert \tilde{u}_{0}\Vert \leq T\), and so \(\tilde{u}_{0}\) is also a critical point of I. For this purpose, we use the following theorem given in [1].

Lemma 2.3

see[1]

Let X be a Banach space with norm \(\Vert \cdot \Vert _{X}\), and \(K\subset\mathbb{R}^{+}\) be an interval. Consider the family of \(C^{1}\) functionals on X

$$ I_{\mu}(u)=A(u)-\mu B(u),\quad \mu\in K, $$
(2.28)

with B nonnegative and either \(A(u)\rightarrow\infty\) or \(B(u)\rightarrow\infty\) as \(\Vert u\Vert _{X}\rightarrow\infty\) and \(I_{\mu}(0)=0\). For any \(\mu\in K\), we set

$$ \Gamma_{\mu}= \bigl\{ \gamma\in \bigl( C[0,1], X \bigr): \gamma(0)=0,I_{\mu}\bigl(\gamma(1) \bigr)< 0 \bigr\} . $$
(2.29)

If for any \(\mu\in K\), the set \(\Gamma_{\mu}\) is nonempty, and

$$ c_{\mu}=\inf_{\gamma\in \Gamma_{\mu}}\max_{t\in[0,1]}I_{\mu}\bigl(\gamma(t) \bigr)>0, $$
(2.30)

then, for almost every \(\mu\in K\), there is a sequence \(\{u_{n}\}\subset X\) such that (i) \(\{u_{n}\}\) is bounded; (ii) \(I_{\mu}(u_{n})\rightarrow c_{\mu}\); (iii) \(I'_{\mu}(u_{n})\rightarrow0\) in \(X^{-1}\).

In our case,

$$ A(u)=\frac{a}{p}\Vert u\Vert ^{p}+\frac{\lambda}{p(m+1)} \eta_{T}(u)\Vert u\Vert ^{p(m+1)}, \quad\quad B(u)= \int_{\mathbb{R}^{N}} \bigl(F(u)+hu \bigr)\,dx. $$
(2.31)

So the perturbed functional we study is

$$ I^{T}_{\mu}(u)=\frac{a}{p}\Vert u\Vert ^{p}+\frac{\lambda}{p(m+1)}\eta_{T}(u)\Vert u\Vert ^{p(m+1)}-\mu \int_{\mathbb{R}^{N}} \bigl(F(u)+hu \bigr)\,dx, $$
(2.32)

and

$$ \bigl(I^{T}_{\mu}(u) \bigr)'v={\widehat{M}} \bigl(\Vert u\Vert \bigr) \int_{\mathbb{R}^{N}} \bigl(\vert \nabla{u} \vert ^{p - 2} \nabla{u}\nabla{v} + \vert u\vert ^{p - 2}uv \bigr)\,dx -\mu \int_{\mathbb{R}^{N}} \bigl(f(u)+h \bigr)v\,dx, $$
(2.33)

where \({\widehat{M}}(\Vert u\Vert )=(a + \lambda\eta_{T}(u)\Vert u\Vert ^{pm} + \frac{\lambda}{(m + 1)T^{p}}\eta'_{T}(u)\Vert u\Vert ^{p(m + 1)})\). The following lemmas, Lemma 2.4 and Lemma 2.5, imply that \(I^{T}_{\mu}\) satisfies the conditions of Lemma 2.3.

Lemma 2.4

Let \((F_{1})\)-\((F_{3})\) hold, Then \(\Gamma_{\mu}\neq\emptyset\) for all \(\mu\in[\frac{1}{2},1]\).

Proof

Choose \(\beta(x)\in C_{0}^{1}(\mathbb{R}^{N})\) with \(\beta(x)\geq0\) in \(\mathbb{R}^{N}\), \(\Vert \beta \Vert =1\), and \(\operatorname{supp}(\beta)\subset B_{R}\) for some \(R>0\). By \((F_{3})\) we have that, for any \(C_{3}>0\) with \({C_{3}}/{2}\int_{B_{R}}\beta^{p}\,dx>{a}/{p}\), there exists \(C_{4}>0\) such that

$$ F(t)\geq C_{3}\vert t\vert ^{p}-C_{4}, \quad t\in \mathbb{R}^{+}. $$
(2.34)

Then, for \(t^{p}>2T^{p}\),

$$ \begin{aligned}[b] I^{T}_{\mu}(t\beta)&= \frac{a}{p}\Vert t\beta \Vert ^{p}+\frac{\lambda}{p(m+1)}\psi \biggl( \frac{\Vert t\beta \Vert ^{p}}{T^{p}} \biggr)\Vert t\beta \Vert ^{p(m+1)}-\mu \int_{\mathbb{R}^{N}} \bigl(F(t\beta)+ht\beta \bigr)\,dx \\ &=\frac{a}{p}\Vert t\beta \Vert ^{p}-\mu \int_{\mathbb{R}^{N}} \bigl(F(t\beta)+ht\beta \bigr)\,dx\leq \biggl( \frac{a}{p} -\frac{C_{3}}{2} \int_{B_{R}}\beta^{p}\,dx \biggr)t^{p}+C_{5}. \end{aligned} $$
(2.35)

It follows that we can choose \(t>0\) large enough such that \(I^{T}_{\mu}(t\beta)<0\). The proof is completed. □

Lemma 2.5

Let \((F_{1})\)-\((F_{3})\) hold. Then there exists a constant \(c>0\) such that \(c_{\mu}\geq c>0\) for all \(\mu\in[\frac{1}{2},1]\) if \(\Vert h\Vert _{p'}< m_{1}\).

Proof

Similarly as in the proof of Lemma 2.1, we can show that, for every \(\mu\in[\frac{1}{2},1]\), there exists \(c>0\) such that \(I^{T}_{\mu}(u)\geq c\) with \(\Vert u\Vert =\tilde{\rho}\) and \(\Vert h\Vert _{p'}< m_{1}\). Fix \(\mu\in[\frac{1}{2},1]\) and \(\gamma\in\Gamma_{\mu}\). By the definition of \(\Gamma_{\mu}\), \(\Vert \gamma({1})\Vert >\tilde{\rho }\). By the continuity we deduce that there exists \(t_{\gamma}\in(0,1)\) such that \(\Vert \gamma({t_{\gamma}})\Vert _{E}=\tilde{\rho}\). Therefore, for any \(\mu\in[\frac{1}{2},1]\),

$$ c_{\mu}=\inf_{\gamma\in\Gamma_{\mu}}\max_{t\in[0,1]}I^{T}_{\mu}\bigl( \gamma(t) \bigr)\geq\inf_{\gamma\in\Gamma_{\mu}} I^{T}_{\mu}\bigl( \gamma(t_{\gamma}) \bigr)\geq c>0, $$
(2.36)

which completes the proof. □

Lemma 2.6

For any \(\mu\in[\frac{1}{2},1]\) and \(a>2^{m+1}(\frac{m+3}{m+1})\lambda T^{pm}\), each bounded (PS) sequence of the functional \(I^{T}_{\mu}\) admits a convergent subsequence.

Proof

By Lemmas 2.3-2.5, we obtain that, for a.e. \(\mu\in [1/2,1]\), there is a bounded sequence \(\{u_{n}\}\) in X that satisfies

$$ I^{T}_{\mu}(u_{n})\rightarrow c_{\mu},\quad\quad \bigl(I^{T}_{\mu}(u_{n}) \bigr)' \rightarrow0 \quad \text{in } X^{*},\quad \text{and} \quad\sup_{n} \Vert u_{n}\Vert < T. $$
(2.37)

Since the embedding \(X\hookrightarrow L^{q}(\mathbb{R}^{N})\) is compact for \(q\in(p,p^{*})\), passing to a subsequence, we can assume that

$$ u_{n}\rightharpoonup u, \quad\text{in } X; \quad\quad u_{n} \rightarrow u, \quad L^{q} \bigl(\mathbb{R}^{N} \bigr);\quad\quad u_{n} \rightarrow u, \quad \text{a.e. in } \mathbb{R}^{N}. $$
(2.38)

By (2.16) we can get

$$ \bigl(I^{T}_{\mu}(u_{n})-I^{T}_{\mu}(u) \bigr)'(u_{n}-u)=A_{n}+B_{n}+\mu C_{n}, $$
(2.39)

where

$$ \begin{gathered} A_{n}= \widehat{M}(u_{n}) \int_{\mathbb{R}^{N}} \bigl(\vert \nabla u_{n}\vert ^{p-2}\nabla u_{n} -\vert \nabla u\vert ^{p-2} \nabla u \bigr)\nabla(u_{n}-u) \\ \hphantom{A_{n}=}{}+ \bigl(\vert u_{n}\vert ^{p-2}u_{n}-u^{p-2}u \bigr) (u_{n}-u)\,dx, \\ B_{n}= \bigl(\widehat{M}(u_{n})-\widehat{M}(u) \bigr) \int_{\mathbb{R}^{N}}\vert \nabla u\vert ^{p-2}\nabla{u} \nabla(u_{n}-u)+\vert u\vert ^{p-2}u(u_{n}-u)\,dx, \\ C_{n}= \int_{\mathbb{R}^{N}} \bigl(f(u_{n})-f(u) \bigr) (u_{n}-u)\,dx. \end{gathered} $$
(2.40)

It is clear that

$$ \bigl(I^{T}_{\mu}(u_{n})-I^{T}_{\mu}(u) \bigr)'(u_{n}-u)\rightarrow0 \quad\text{as } n \rightarrow \infty. $$
(2.41)

An analogous argument as in (2.22) and (2.25) gives us that

$$ B_{n}\to0 \quad \mbox{and}\quad C_{n}\to0 \quad\text{as } n\to\infty. $$
(2.42)

Combining the above results and \(a>2^{m+1}(\frac{m+3}{m+1})\lambda T^{pm}\), we have that \(\vert A_{n}\vert \to0\) as \(n\to\infty\). Then, using a standard equality ([3], Lemma 2.1), we can show that \(u_{n} \rightarrow u\) in X. The proof is completed. □

Lemma 2.7

Assume \((F_{1})\)-\((F_{3})\) and \(a>2^{m+1}(\frac{m+3}{m+1})\lambda T^{pm}\). Then, for almost every \(\mu\in[\frac{1}{2},1]\), there exist \(u^{\mu}\in X\setminus\{0\}\) such that \((I^{T}_{\mu})'(u^{\mu})=0\) and \(I^{T}_{\mu}(u^{\mu})=c_{\mu}\) with \(\Vert h\Vert _{p'}< m_{1}\).

Proof

It follows from Lemmas 2.3-2.5 that, for every \(\mu \in[\frac{1}{2},1]\), there exists a bounded sequence \(\{u_{n}^{\mu}\} \subset X\) such that

$$ I^{T}_{\mu}\bigl(u^{\mu}_{n} \bigr)\rightarrow c_{\mu}\quad \mbox{and} \quad \bigl(I^{T}_{\mu}\bigr)' \bigl(u^{\mu}_{n} \bigr)\rightarrow0\quad \mbox{as } n\rightarrow\infty. $$

By Lemma 2.6 we can suppose that \(u^{\mu}\in X\) and \(u^{\mu}_{n}\rightarrow u^{\mu}\) in X. The proof is completed. □

According to Lemma 2.6, there exists a sequence \(\{\mu_{n}\}\subset[\frac{1}{2},1]\) with \(\mu_{n}\rightarrow1\) and \(\{u_{n}\}\subset X\) as \(n\rightarrow\infty\) such that \(I^{T}_{\mu_{n}}(u_{n})=c_{\mu_{n}}\), \((I^{T}_{\mu_{n}})'(u_{n})=0\), and \(u_{n}\) is a positive solution of

$$\begin{aligned} {\widehat{M}} \bigl(\Vert u\Vert \bigr) \bigl(-\Delta_{p} u+ \vert u\vert ^{p-2}u \bigr)=\mu_{n} \bigl(f(u)+h(x) \bigr). \end{aligned}$$
(2.43)

In the following, to obtain \(\Vert u_{n}\Vert < T\), we establish an identity that extends the Kazin-Pohozav identity in ([13], Thm. 29.4) with \(p=2\).

Lemma 2.8

Assume that \(f(x,u):\mathbb{R}^{N}\times\mathbb{R}^{1}\rightarrow\mathbb{R}^{1}\) is a Carethéodary function, \(u\in C^{2}_{\mathrm {loc}}(\mathbb{R}^{N})\) is a solution of

$$ \textstyle\begin{cases} -\Delta_{p} u+f(x,u)=0\quad \textit{in } \mathbb{R}^{N}, \\ u(x)\rightarrow0 \quad\textit{as } \rightarrow0, \end{cases} $$
(2.44)

\(\frac{\partial u}{\partial x_{i}}\in L^{p}({\mathbb{R}^{N}})\), \(i=1,2,\ldots\) , and \(F(x,u), F_{1}(x,u)\in L^{1}(\mathbb{R}^{N})\). Then

$$ \frac{N-p}{p} \int_{\mathbb{R}^{N}}\vert \nabla u\vert ^{p}\,dx+ \int_{\mathbb{R}^{N}} \bigl(NF(x,u)+F_{1}(x,u) \bigr)\,dx=0, $$
(2.45)

where \(F(x,u)=\int^{u}_{0}f(x,s)\,ds\) and \(F_{1}(x,u)=\sum^{N}_{i=1}x_{i}\frac{\partial F(x,u)}{\partial x_{i}}\).

Proof

Multiplying equation (2.44) by \(x\cdot\nabla u\) and integrating over the ball \(B_{R}\), we obtain

$$\begin{aligned} \int_{B_{R}}f(x,u)x\cdot\nabla u\,dx= \int_{B_{R}}\operatorname {div}\bigl(\vert \nabla u\vert ^{p-2} \nabla u \bigr)x\cdot\nabla u\,dx. \end{aligned}$$
(2.46)

Then

$$ \begin{aligned}[b] \int_{B_{R}}f(x,u)x\cdot\nabla u\,dx&=\sum ^{N}_{i=1} \int_{B_{R}}x_{i} f(x,u)\frac{\partial u}{\partial x_{i}}\,dx \\ &=\sum^{N}_{i=1} \int_{B_{R}} \biggl(\frac{\partial}{\partial x_{i}} \bigl(x_{i}F(x,u) \bigr)- \biggl(F(x,u)+x_{i}\frac{\partial F(x,u)}{\partial x_{i}} \biggr) \biggr)\,dx \\ &=\sum^{N}_{i=1} \int_{\partial B_{R}}F(x,u) x_{i}n_{i}\,ds- \int_{B_{R}} \bigl(NF(x,u)+F_{1}(x,u) \bigr)\,dx \\ &=R \int_{\partial B_{R}}F(x,u)\,ds- \int_{B_{R}} \bigl(NF(x,u)+F_{1}(x,u) \bigr)\,dx, \end{aligned} $$
(2.47)

where \(n_{i}\) are the components of the unit outward normal to \(\partial B_{R}\), and ds is an area element. On the other hand, integrating by parts, we obtain

$$ \begin{aligned}[b] & \int_{B_{R}}\operatorname {div}\bigl(\vert \nabla u\vert ^{p-2} \nabla u \bigr)x\cdot\nabla u\,dx \\ &\quad =\sum^{N}_{j=1} \int_{B_{R}}\frac{\partial}{\partial x_{j}} \biggl(\vert \nabla u\vert ^{p-2}\frac{\partial u}{\partial x_{j}} \biggr)\sum^{N}_{i=1}x_{i} \frac{\partial u}{\partial x_{i}}\,dx \\ &\quad =\sum^{N}_{j=1} \int_{B_{R}} \Biggl(\frac{\partial}{\partial x_{j}} \Biggl(\vert \nabla u \vert ^{p-2}\frac{\partial u}{\partial x_{j}}\sum^{N}_{i=1}x_{i} \frac{\partial u}{\partial x_{i}} \Biggr)-\vert \nabla u\vert ^{p-2} \frac{\partial u}{\partial x_{j}} \Biggl(\frac{\partial}{\partial x_{j}}\sum^{N}_{i=1}x_{i} \frac{\partial u}{\partial x_{i}} \Biggr) \Biggr)\,dx \\ &\quad = \int_{\partial B_{R}} \vert \nabla u\vert ^{p-2} \frac{\partial u}{\partial n}x\cdot\nabla u\,ds- \int_{B_{R}}\vert \nabla u\vert ^{p}\,dx \\ &\quad\quad{} - \int_{B_{R}}\sum^{N}_{j=1} \vert \nabla u\vert ^{p-2} \Biggl(\sum^{N}_{i=1}x_{i} \frac{\partial^{2} u}{\partial x_{i}\,\partial x_{j}}\frac{\partial u}{\partial x_{j}} \Biggr)\,dx. \end{aligned} $$
(2.48)

On \(B_{R}\), we have \(\nabla u=\frac{\partial u}{n}\cdot\vec{n}=\frac {\partial u}{\partial n}\frac{x}{R}\) and

$$ \int_{\partial B_{R}}\vert \nabla u\vert ^{p-2} \frac{\partial u}{\partial n}x\cdot\nabla u\,dx=R \int_{\partial B_{R}}\vert \nabla u\vert ^{p}\,ds. $$
(2.49)

Further, we have

$$ \begin{aligned}[b] & \int_{B_{R}}\sum^{N}_{j=1} \vert \nabla u\vert ^{p-2} \Biggl(\sum^{N}_{i=1}x_{i} \frac{\partial^{2} u}{\partial x_{i}\,\partial x_{j}}\frac{\partial u}{\partial x_{j}} \Biggr)\,dx \\ &\quad =\frac{1}{p}\sum^{N}_{i=1} \int_{B_{R}} \biggl(\frac{\partial}{\partial x_{i}} \bigl(x_{i} \vert \nabla u\vert ^{p} \bigr)-\vert \nabla u\vert ^{p} \biggr) \,dx \\ &\quad =\frac{R}{p} \int_{\partial B_{R}}\vert \nabla u\vert ^{p}\,ds- \frac{N}{p} \int_{B_{R}}\vert \nabla u\vert ^{p}\,dx. \end{aligned} $$
(2.50)

Therefore, we obtain

$$ R \int_{\partial B_{R}} \biggl(F- \biggl(1-\frac{1}{p} \biggr)\vert \nabla u\vert ^{p} \biggr)\,ds+ \biggl(1-\frac{N}{p} \biggr) \int_{B_{R}}\vert \nabla u\vert ^{p}\,dx- \int_{B_{R}}(NF+F_{1})\,dx=0. $$
(2.51)

Since \(F(x,u)\in L^{1}({\mathbb {R}}^{N})\) and \(u\in X\), we claim that

$$ \liminf_{n\to\infty}R \int_{\partial B_{R}} \bigl( \bigl\vert F(x,u) \bigr\vert +\vert \nabla u\vert ^{p} \bigr)\,dS=0. $$
(2.52)

Indeed, otherwise,

$$ \liminf_{n\to\infty}R \int_{\partial B_{R}} \bigl( \bigl\vert F(x,u) \bigr\vert +\vert \nabla u\vert ^{p} \bigr)\,dS=a_{0}>0. $$
(2.53)

Then, there exists \(R_{0}>0\) such that, for \(R\geq R_{0}\),

$$ R \int_{\partial B_{R}} \bigl( \bigl\vert F(x,u) \bigr\vert +\vert \nabla u\vert ^{p} \bigr)\,dS\geq\frac{a_{0}}{2}. $$
(2.54)

Let \(R_{n}=R_{0}+n\), \(n=1,2,\dots\). Then \(R_{n}\to\infty\) as \(n\to\infty\). It follows from the integral mean theorem that there is \(\xi_{n}\in (R_{n-1},R_{n})\) and \(\xi_{n}\geq R_{0}\) such that, for \(R\geq R_{0}\),

$$ \int^{R_{n}}_{R_{n-1}} \int_{\partial B_{R}} \bigl(\vert F\vert +\vert \nabla u\vert ^{p} \bigr)\,ds\,dR=\xi_{n} \int_{\partial B_{\xi_{n}}} \bigl(\vert F\vert +\vert \nabla u\vert ^{p} \bigr)\,ds\geq\frac{a_{0}}{2}, $$
(2.55)

and thus

$$ \int^{\infty}_{R_{0}} \int_{\partial B_{R}} \bigl(\vert F\vert +\vert \nabla u\vert ^{p} \bigr)\,ds\,dR\geq\sum^{\infty}_{n=2} \int^{R_{n}}_{R_{n-1}} \int_{\partial B_{R}} \bigl(\vert F\vert +\vert \nabla u\vert ^{p} \bigr)\,ds\,dR=\infty. $$
(2.56)

This contradicts the fact

$$ \int_{{\mathbb {R}}^{N}} \bigl(\vert F\vert +\vert \nabla u\vert ^{p} \bigr)\,dx= \int^{\infty}_{0} \int_{\partial B_{R}} \bigl(\vert F\vert +\vert \nabla u\vert ^{p} \bigr)\,ds\,dR< \infty. $$
(2.57)

Therefore, (2.52) is true. Thus, letting \(R\to\infty\) in (2.51), we have

$$ \frac{N-p}{p} \int_{\mathbb{R}^{N}}\vert \nabla u\vert ^{p}\,dx+ \int_{\mathbb{R}^{N}} \bigl(NF(x,u)+F_{1}(x,u) \bigr)\,dx=0. $$
(2.58)

Then, we finish the proof of Lemma 2.8. □

Lemma 2.9

Let \(a>2^{m+1}(\frac{m+3}{m+1})\lambda T^{pm}\), and let \(u\in X\) be a weak solution of

$$ {\widehat{M}} \bigl(\Vert u\Vert \bigr) \bigl(- \Delta_{p} u+\vert u\vert ^{p-2}u \bigr)=\mu \bigl(f(u)+h(x) \bigr), $$
(2.59)

where \({\widehat{M}}(\Vert u\Vert )=(a + \lambda\eta_{T}(u)\Vert u\Vert ^{pm} + \frac{\lambda}{(m + 1)T^{p}}\eta'_{T}(u)\Vert u\Vert ^{p(m + 1)})\). Then the following identity holds:

$$ \begin{aligned}[b] &{\widehat{M}} \bigl(\Vert u\Vert \bigr) \biggl( \frac{N-p}{p} \int_{\mathbb{R}^{N}} \vert \nabla u\vert ^{p}\,dx + \frac{N}{p} \int_{\mathbb{R}^{N}}\vert u\vert ^{p}\,dx \biggr) \\ &\quad = N\mu \int_{\mathbb{R}^{N}} \bigl(F(u)+hu \bigr)\,dx + \mu \int_{\mathbb{R}^{N}}\nabla h\cdot xu\,dx. \end{aligned} $$
(2.60)

Proof

Since \(u\in X\) is a weak solution of (2.59), by standard regularity results, \(u\in C^{2}_{\mathrm {loc}}(\mathbb{R}^{N})\cap W^{1,p}(\mathbb {R}^{N})\). Let

$$ g(x,u)=\frac{\mu(f(u)+h(x))}{{\widehat{M}}(\Vert u\Vert )}-\vert u\vert ^{p-2}u. $$
(2.61)

Then \(u\in X\) is also a solution of

$$ -\Delta_{p} u=g(x,u). $$
(2.62)

By Lemma 2.8,

$$ \frac{N-p}{p} \int_{\mathbb{R}^{N}}\vert \nabla u\vert ^{p}\,dx= \int_{\mathbb{R}^{N}} \bigl(NG(u)+ G_{1}(x, u) \bigr)\,dx, $$
(2.63)

where \(G(x,u)=\int^{u}_{0}g(x,s)\,ds\) and \(G_{1}(x,u)=\sum^{N}_{i=1}x_{i}\frac {\partial G(x,u)}{\partial x_{i}}\). Then the conclusion holds. □

Lemma 2.10

Assume that \((F_{1})\)-\((F_{3})\) and \((H)\) hold and that \(\Vert h\Vert _{p'}< m_{1}\) for \(m_{1}\) given in Lemma  2.6. Let \(u_{n}\) be a critical point of \(I^{T}_{\mu_{n}}\) at level \(c_{\mu_{n}}\). Then for T sufficiently large, there exists \(\lambda_{0}=\lambda_{0}(T)\) with \(\lambda_{0}< a(\frac{m+1}{m+3})T^{-pm}\) such that, for any \(\lambda\in[0,\lambda_{0})\), subject to a subsequence, \(\Vert u_{n}\Vert < T\) for all \(n\in\mathbb{N}\).

Proof

Since \((I^{T}_{\mu_{n}})'(u_{n})=0\), by Lemma 2.9 \(u_{n}\) satisfies

$$ \begin{aligned}[b] & {\widehat{M}} \bigl( \Vert u\Vert \bigr) \biggl( \frac{N}{p}\Vert u\Vert ^{p}+ \int_{{\mathbb {R}}^{N}}\vert \nabla u\vert ^{p}\,dx \biggr) \\ &= N \mu_{n} \int_{\mathbb{R}^{N}} \bigl(F(u_{n}) + hu_{n} \bigr)\,dx + \mu_{n} \int_{\mathbb{R}^{N}} \nabla h\cdot xu_{n}\,dx. \end{aligned} $$
(2.64)

Using \(I^{T}_{\mu_{n}}(u_{n})=c_{\mu_{n}}\), we have

$$ \frac{aN}{p}\Vert u_{n}\Vert ^{p}+ \frac{\lambda N}{p(m+1)}\eta_{T}(u_{n})\Vert u_{n} \Vert ^{p(m+1)}=N\mu_{n} \int_{\mathbb{R}^{N}} \bigl(F(u_{n})+h u_{n} \bigr)\,dx+Nc_{\mu_{n}}. $$
(2.65)

Therefore, by (2.64), (2.65) and \(a>2^{m+1}(\frac {m+3}{m+1})\lambda T^{pm}\) we deduce that

$$\begin{aligned}& \frac{a}{2} \int_{\mathbb{R}^{N}}\vert \nabla u_{n}\vert ^{p}\,dx \\& \quad \leq{\widehat{M}} \bigl( \Vert u_{n}\Vert \bigr) \int_{\mathbb{R}^{N}}\vert \nabla u_{n}\vert ^{p}\,dx \\& \quad =N c_{\mu_{n}}+N \biggl(\widehat{M} \bigl( \Vert u_{n} \Vert \bigr)-\frac{a}{p} \biggr)\Vert u\Vert ^{p}- \frac{\lambda N}{p(m+1)}\eta_{T}(u_{n})\Vert u_{n} \Vert ^{p(m+1)} - \mu_{n} \int_{\mathbb{R}^{N}}\nabla hx\cdot u_{n}\,dx \\& \quad \leq N c_{\mu_{n}} + \frac{\lambda Nm}{p(m+1)}\eta_{T}(u_{n}) \Vert u_{n}\Vert ^{p(m+1)} + \frac{\lambda N}{p(m+1)T^{p}} \eta'_{T}(u_{n})\Vert u_{n}\Vert ^{p(m+2)} \\& \quad{} - \mu_{n} \int_{\mathbb{R}^{N}}\nabla hx\cdot u_{n}\,dx. \end{aligned}$$
(2.66)

By the min-max definition of the mountain pass level, Lemma 2.5, and (2.35) we have

$$ \begin{aligned}[b] c_{\mu_{n}}&\leq\max_{t} I^{T}_{\mu_{n}}(t \beta) \\ &\leq\max_{t} \biggl\{ \biggl(\frac{a}{p}- \frac{C_{3}}{2} \int_{B_{R}}\vert \beta \vert ^{p}\,dx \biggr)t^{p}+C_{5} \biggr\} +\max_{t} \frac{\lambda}{p(m+1)} \psi \biggl(\frac{t^{p}}{T^{p}} \biggr)t^{p(m+1)} \\ &\leq\frac{\lambda2^{m+1}}{p(m+1)}T^{p(m+1)}+C_{5}. \end{aligned} $$
(2.67)

Using \((H)\) and the Young equality, we have

$$ \begin{aligned}[b] \int_{\mathbb{R}^{N}}\nabla h\cdot xu_{n} \,dx &\leq\frac{ 1}{p'} \int_{\mathbb{R}^{N}}\vert \xi \vert ^{p'}\,dx+ \frac{1}{p} \int_{\mathbb{R}^{N}}\vert \xi \vert ^{p'}\vert u_{n}\vert ^{p}\,dx \\ & \leq\frac{1}{p} \int_{\mathbb{R}^{N}}\vert \xi \vert ^{p'}\vert u_{n}\vert ^{p}\,dx+C_{6}. \end{aligned} $$
(2.68)

We can easily calculate that

$$ \eta_{T}(u_{n})\Vert u_{n} \Vert ^{p(m+1)}\leq2^{m+1}T^{p(m+1)}, \quad\quad \eta'(u_{n}) \Vert u_{n}\Vert ^{p(m+2)}\leq2^{m+2}T^{p(m+2)}. $$
(2.69)

Combining the above estimates, we see that

$$ \frac{a}{2} \int_{\mathbb{R}^{N}}\vert \nabla u_{n}\vert ^{p}\,dx \leq\frac{\lambda N(m+5)}{p(m+1)}2^{m+1}T^{p(m+1)}+ \frac{1}{p} \int_{\mathbb{R}^{N}}\vert \xi \vert ^{p'}\vert u_{n}\vert ^{p}\,dx+C_{7}. $$
(2.70)

Since \(\xi(x)\in L^{p'}(\mathbb{R}^{N})\cap W^{1,\infty}\), we see that \(\xi^{p'}u_{n}\in X\). It follows from \((I^{T}_{\mu_{n}}(u_{n}))'(\xi^{p'}u_{n})=0\) that

$$ \begin{aligned}[b] & \widehat{M} \bigl( \bigl\Vert \xi^{p'}u_{n} \bigr\Vert \bigr) \int_{\mathbb{R}^{N}}\vert \nabla u_{n}\vert ^{p-2} \nabla u_{n}\nabla \bigl(\xi^{p'}u_{n} \bigr) + \vert u_{n}\vert ^{p-2}u \bigl(\xi^{p'}u_{n} \bigr)\,dx \\ &\quad = \mu_{n} \int_{\mathbb{R}^{N}} \bigl(f(u_{n})+h \bigr) \xi^{p'}u_{n}\,dx. \end{aligned} $$
(2.71)

Since \(a>2^{m+1}(\frac{m+3}{m+1})\lambda T^{pm}\), we have \(({3a}/2)\geq \widehat{M}( \Vert \xi^{p'}u_{n}\Vert )\), and it follows from (2.69) and (2.71) that

$$ ({3a}/2) \int_{\mathbb{R}^{N}}\vert \nabla u_{n}\vert ^{p-2} \nabla u_{n}\nabla \bigl(\xi^{p'}u_{n} \bigr) +\vert u_{n}\vert ^{p}\xi^{p'}\,dx\geq(1/ {2}) \int_{\mathbb{R}^{N}}f(u_{n})u_{n} \xi^{p'}\,dx. $$
(2.72)

From (2.70) by the Hölder inequality we deduce that

$$ \begin{aligned}[b] &{3a} \int_{\mathbb{R}^{N}}\vert \nabla u_{n}\vert ^{p-2} \nabla u_{n}\nabla \bigl(\xi^{p'}u_{n} \bigr)\,dx \\ &\quad \leq{3a} \int_{\mathbb{R}^{N}}\vert \nabla u_{n}\vert ^{p-2} \nabla u_{n} \bigl(p'\xi^{p'-1}u_{n} \nabla\xi+\xi^{p'}\nabla u_{n} \bigr)\,dx \\ &\quad \leq3 \bigl(\Vert \xi \Vert ^{\infty}_{p'}+\Vert \nabla \xi \Vert ^{\infty}_{p'} \bigr) \biggl({a} \int_{\mathbb{R}^{N}}\vert \nabla u_{n}\vert ^{p}\,dx \biggr)+{3a} {(p-1)^{-1}} \int_{\mathbb{R}^{N}}\xi^{p'}\vert u_{n}\vert ^{p}\,dx \\ &\quad \leq C\lambda T^{p(m+1)}+C \int_{\mathbb{R}^{N}}\xi^{p'}\vert u_{n}\vert ^{p}\,dx+C, \end{aligned} $$
(2.73)

where C is a constant independent of λ and T.

By \((F_{3})\), for any \(L>0\), there exists \(C(L)>0\) such that

$$ f(s)s\geq Ls^{p}-C(L) \quad\text{for all } s>0. $$
(2.74)

Combining (2.72)-(2.74), we get

$$ \biggl(\frac{1}{2}L-C \biggr) \int_{\mathbb{R}^{N}}\xi^{p'}\vert u_{n}\vert ^{p}\,dx\leq C\lambda T^{p(m+1)}+C. $$
(2.75)

For \(L>0\) large enough, we obtain

$$ \int_{\mathbb{R}^{N}}\xi^{p'}\vert u_{n}\vert ^{p}\,dx\leq C\lambda T^{p(m+1)}+C. $$
(2.76)

It follows from (2.70) and (2.76) that

$$ \int_{\mathbb{R}^{N}}\vert \nabla u_{n}\vert ^{p}\,dx \leq C\lambda T^{p(m+1)}+C. $$
(2.77)

On the other hand,

$$ \begin{aligned}[b] &a\Vert u_{n}\Vert ^{p} + \eta_{T}(u_{n})\Vert u_{n} \Vert ^{p(m+1)} + \frac{\lambda}{m+1}\eta'_{T}(u_{n}) \Vert u_{n}\Vert ^{p(m+2)} \\ &\quad = \mu_{n} \int_{\mathbb{R}^{N}} \bigl(f(u_{n})u_{n}+hu_{n} \bigr)\,dx \\ &\quad \leq\varepsilon \Vert u_{n}\Vert ^{p} + C_{\varepsilon} \Vert u_{n}\Vert _{p^{*}}^{p^{*}} + \frac{1}{p'} \Vert h\Vert ^{p'}_{p'} + \frac{1}{p}\Vert u \Vert ^{p}. \end{aligned} $$
(2.78)

By (2.77) and (2.78) we have

$$ \begin{aligned}[b] (a-\varepsilon-{1}/{p})\Vert u_{n}\Vert ^{p}&\leq C_{\varepsilon} \Vert u_{n}\Vert ^{p*}_{p*}-{\lambda}/ \bigl({(m+1)T^{p}} \bigr) \eta'_{T}(u_{n})\Vert u_{n}\Vert ^{p(m+2)}+C \\ &\leq C\Vert \nabla u_{n}\Vert ^{p*}_{p}+{ \lambda2^{m+2}} {(m+1)}^{-1}T^{p(m+1)}+C \\ &\leq C\lambda T^{p^{*}(m+1)}+C\lambda T^{p(m+1)}+C. \end{aligned} $$
(2.79)

Suppose that \(\Vert u_{n}\Vert >T\) for \(n\in {\mathbb {N}}\) and T large enough. Then

$$ T^{p}< \Vert u_{n}\Vert ^{p}\leq C\lambda T^{p^{*}(m+1)}+C\lambda T^{p(m+1)}+C, $$
(2.80)

which is not true if we choose T large and λ small enough. So by setting \(\lambda(T)\) small we obtain that the sequence \(\{u_{n}\}\) is bounded for any \(\lambda\in[0,\lambda_{0})\), and the conclusion holds. □

Lemma 2.11

Let T, \(\lambda_{0}\) be defined by Lemma  2.10, and \(u_{n}\) be the critical point of \(I^{T}_{\mu_{n}}\) at level \(c_{\mu_{n}}\). Then the sequence \(\{u_{n}\}\) is also a (PS) sequence for I.

Proof

From the proof of Lemma 2.10 we may assume that \(\Vert u_{n}\Vert \leq T\). So

$$ I(u_{n})=I^{T}_{\mu_{n}}(u_{n})+( \mu_{n}-1) \int_{\mathbb{R}^{N}} \bigl(F(u_{n})+hu_{n} \bigr)\,dx. $$
(2.81)

Since \(\mu_{n}\rightarrow1\), we can show that \(\{u_{n}\}\) is a (PS) sequence of I. Indeed, the boundedness of \(\{u_{n}\}\) implies that \(\{I^{T}_{\mu_{n}}\}\) is bounded. Also,

$$ I'(u_{n})v= \bigl(I^{T}_{\mu_{n}} \bigr)'(u_{n},v)+(\mu_{n}-1) \int_{\mathbb{R}^{N}} \bigl(f(u_{n})+h(u_{n}) \bigr)v\,dx, \quad v\in X. $$
(2.82)

Thus, \(I'(u_{n})\rightarrow0\), and \(\{u_{n}\}\) is a bounded (PS) sequence of I. By Lemma 2.5, \(\{u_{n}\}\) has a convergent subsequence. We may assume that \(u_{n}\rightarrow\tilde{u}_{0}\). Consequently, \(I'(\tilde{u}_{0})=0\). According to Lemma 2.4, we have that \(I(\tilde{u}_{0})=\lim_{n\rightarrow\infty} I(u_{n})=\lim_{n\rightarrow\infty}I^{T}_{\mu_{n}}(u_{n})\geq c>0\) and \(\tilde{u}_{0}\) is a solution of problem (1.1). Thus, we completed the proof. □

Proof of Theorem 1.1

By Lemma 2.2 the problem has a solution \(u_{0}\in X\) with \(I(u_{0})<0\). From Lemma 2.9 we know that problem (1.1) possesses a second solution \(\tilde{u}_{0}\in X\) with \(I(\tilde{u}_{0})\geq c>0\). Hence, \(u_{0}\neq\tilde{u}_{0}\), and we complete the proof of Theorem 1.1. □

3 Proof of Theorem 1.2

Let \(I_{\lambda}(u):X\rightarrow\mathbb{R} \) be the energy functional associated with problem (1.13) defined by

$$ I_{\lambda}(u)=\frac{a}{p}\Vert u\Vert ^{p}+ \frac{\lambda}{p(m+1)}\Vert u\Vert ^{p(m+1)}-\frac{1}{q}\Vert u \Vert ^{q}_{q}, $$
(3.1)

where \(F(u)=\int_{0}^{u}f(s)\,ds\). It is easy to see that the functional \(I\in C^{1}(E,\mathbb{R})\) and its Gateaux derivative is given by

$$ \begin{aligned}[b] I_{\lambda}'(u)v&= \bigl(a+\lambda \Vert u\Vert ^{pm} \bigr) \int_{\mathbb{R}^{N}} \bigl(\vert \nabla u\vert ^{p-2} \nabla u \nabla v+\vert u\vert ^{p-2}uv \bigr)\,dx \\ &\quad{} - \int_{\mathbb{R}^{N}} \vert u\vert ^{q-2}uv\,dx,\quad \forall v \in E. \end{aligned} $$
(3.2)

Clearly, we see that a weak solution of (1.13) corresponds to a critical point of the functional.

In this part, we first proof the nonexistence for problem (1.13) for large \(\lambda>\lambda^{*}\). which means that if a solution exists, then λ must sufficiently small. Secondly, we obtain that there exists \(\lambda^{**}\) such that problem (1.1) has at least one solution for any \(0<\lambda<\lambda^{**}\). Finally, by the properties of \(\lambda^{*}\) and \(\lambda^{**}\) we deduce that \(\lambda^{*}=\lambda ^{**}\). We will break the proof into six steps.

Proof of Theorem 1.2

Step 1. Nonexistence for large \(\lambda>0\) . It is sufficient to show that if u is a nontrivial solution of problem (1.13), then \(\lambda>0\) must be small. Assume that u is a nontrivial solution of problem (1.1). Then we get \(I'_{\lambda}(u)u=0\), that is,

$$ a\Vert u\Vert ^{p}+\lambda \Vert u\Vert ^{p(m+1)}=\Vert u\Vert _{q}^{q}. $$
(3.3)

Since \(p< q<\min\{p(m+1),p^{*}\}\), applying the Young inequality and (1.9), we deduce that

$$ a\Vert u\Vert ^{p}+\lambda \Vert u\Vert ^{p(m+1)}=\Vert u\Vert _{q}^{q}\leq S_{q}^{q}\Vert u\Vert ^{q}_{E}\leq a \Vert u\Vert ^{p}_{E}+\lambda_{1}\Vert u \Vert ^{p(m+1)}_{E}, $$
(3.4)

which implies that \(\lambda\leq\lambda_{1}=(S^{q}_{q})^{\frac {pm}{q-p}}a^{-\frac{p(m+1)-q}{q-p}}\). On the other hand, if \(\lambda ^{*}\geq\lambda_{1}\), then we conclude that problem (1.1) has no solution for any \(\lambda\in(\lambda^{*}, +\infty)\).

Step 2. Coercivity of \(I_{\lambda}(u)\) . Indeed, for any \(u\in E\) and all \(\lambda>0\),

$$ \begin{aligned}[b] I_{\lambda}(u)&=\frac{a}{p}\Vert u \Vert ^{p}+\frac{\lambda}{p(m+1)}\Vert u\Vert ^{p(m+1)}- \frac{1}{q}\Vert u\Vert ^{q}_{q} \\ &\geq\frac{a}{p}\Vert u\Vert ^{p}+\frac{\lambda}{2p(m+1)} \Vert u\Vert ^{p(m+1)}+\frac{\lambda}{2p(m+1)}\Vert u\Vert ^{p(m+1)}- \frac{S^{q}_{q}}{q}\Vert u\Vert ^{q}. \end{aligned} $$
(3.5)

Since \(q< p(m+1)\), there exists \(C_{1}=C_{1}(\lambda,q,m,S_{q})\) such that

$$ \frac{S^{q}_{q}}{q}\Vert u\Vert ^{q}\leq\frac{\lambda}{2p(m+1)}\Vert u \Vert ^{p(m+1)}+C_{1}. $$
(3.6)

It follows that

$$ I_{\lambda}(u)\geq\frac{a}{p}\Vert u\Vert ^{p}+ \frac{\lambda}{2p(m+1)}\Vert u\Vert ^{p(m+1)}-C_{1}. $$
(3.7)

This implies that \(I_{\lambda}(u)\) is coercive.

Step 3. The infimum of \(I_{\lambda}\) is attained. Let \(\{u_{n}\}\) be a minimizing sequence of \(I_{\lambda}\). Then from Step 2 we immediately see that \(\{u_{n}\}\) is bounded in X. Therefore, without loss of generality, we may assume that \(\{u_{n}\}\) is nonnegative and converges weakly and pointwise to some u in X.

Using the compact embedding \(X\hookrightarrow L^{q}(\mathbb{R}^{N})\), we have

$$ \Vert u\Vert _{q}=\lim_{n\to\infty} \Vert u_{n}\Vert _{q} \quad\text{and} \quad \Vert u\Vert \leq \liminf_{n\to\infty} \Vert u_{n}\Vert $$
(3.8)

by the weak lower semicontinuity of the norm \(\Vert \cdot \Vert \). Thus,

$$\begin{aligned} \begin{aligned}[b] I_{\lambda}(u)&=\frac{a}{p}\Vert u \Vert ^{p}+\frac{\lambda}{p(m+1)}\Vert u\Vert ^{p(m+1)}- \frac{1}{q}\Vert u\Vert ^{q}_{q} \\ &\leq\liminf_{n\to\infty} \biggl(\frac{a}{p}\Vert u_{n}\Vert ^{p}+\frac{\lambda}{p(m+1)}\Vert u_{n} \Vert ^{p(m+1)} \biggr)-\frac{1}{q}\lim_{n\to\infty} \Vert u_{n}\Vert ^{q}_{q} \\ &\leq\liminf_{n\to\infty} \biggl(\frac{a}{p}\Vert u_{n}\Vert ^{p}+\frac{\lambda}{p(m+1)}\Vert u_{n} \Vert ^{p(m+1)}- \frac{1}{q}\Vert u_{n} \Vert ^{q}_{q} \biggr)=\liminf_{n\to\infty}I_{\lambda}(u_{n}). \end{aligned} \end{aligned}$$
(3.9)

Therefore, u is a global minimum for \(I_{\lambda}\), and hence it is a critical point, namely a weak solution to problem (1.1).

Step 4. The weak solution u is nontrivial if \(\lambda>0\) is sufficiently small. Clearly, \(I_{\lambda}(0)=0\). Therefore, it is sufficient to show that there exists \(\lambda_{0}>0\) such that

$$ \inf_{u\in E}I_{\lambda}(u)< 0, \quad \mbox{for any } \lambda\in(0, \lambda_{0}). $$
(3.10)

Choose \(u_{0}\in C^{\infty}_{0}({\mathbb {R}}^{N})\), \(u_{0}\not\equiv0\), such that \(\Vert u_{0}\Vert _{E}=1\). Denote

$$ I_{\lambda}(tu_{0})=t^{p}s(t), \quad\quad s(t)=B_{1}+ \lambda B_{2}t^{pm}-B_{3}t^{q-p}, \quad t\geq0, $$
(3.11)

where

$$B_{1}=\frac{a}{p}, \quad\quad B_{2}=\frac{1}{p(m+1)}>0,\quad\quad B_{3}=\frac{1}{q} \int_{{\mathbb {R}}^{N}}\vert u_{0}\vert ^{q}\,dx>0. $$

Then there exist \(\lambda_{0}>0\) and large \(t_{\lambda}>0\) such that \(I_{\lambda}(t_{\lambda}u_{0})<0\) for \(\lambda\in(0,\lambda_{0}]\). Let \(e=t_{\lambda}u_{0}\). Then \(\Vert e\Vert =t_{\lambda}\) and \(I_{\lambda}(e)<0\). This implies that (3.10) is true. So the weak solution u is nontrivial if \(\lambda>0\) is sufficiently small.

Now, we define

$$\begin{aligned}& \lambda^{**}=\sup \bigl\{ \lambda>0, \mbox{problem (1.13) admits a nontrival weak solution} \bigr\} , \\& \lambda^{*}=\inf \bigl\{ \lambda>0, \mbox{problem (1.13) does not admit any nontrival weak solution} \bigr\} . \end{aligned}$$

Clearly, \(\lambda^{**}\geq\lambda^{*}\). To complete the proof of Theorem 1.2, it suffices to prove the following facts: (a) problem (1.13) has a weak solution for any \(\lambda<\lambda^{**}\); (b) \(\lambda ^{**}=\lambda^{*}\), and problem (1.13) admits a weak solution when \(\lambda=\lambda^{*}\).

Step 5. Problem ( 1.13 ) has a solution for any \(\lambda <\lambda^{**}\) and \(\lambda^{*}=\lambda^{**}\) . Fix \(\lambda<\lambda ^{**}\). By the definition of \(\lambda^{**}\), there exists \(\mu\in (\lambda,\lambda^{**})\) such that \(I_{\lambda}\) has a nontrivial critical point \(u_{\mu}\in E\). Clearly, we have

$$ \biggl(a+\lambda \biggl( \int_{\mathbb{R}^{N}} \bigl(\vert \nabla{u_{m}u}\vert ^{p}+\vert u_{\mu} \vert ^{p} \bigr)\,dx \biggr)^{m} \biggr) \bigl(-\Delta_{p}u_{\mu}+\vert u_{\mu} \vert ^{p-2}u_{\mu}\bigr)\leq \vert u_{\mu} \vert ^{q-2}u_{\mu}. $$
(3.12)

This implies that \(u_{\mu}\) is a subsolution of problem (1.13). In order to find a supsolution of (1.13) that dominates \(u_{\mu}\), we consider the constrained minimization problem

$$ \inf \biggl\{ \frac{a}{p}\Vert \omega \Vert ^{p}+ \frac{\lambda}{p(m+1)}\Vert \omega \Vert ^{p(m+1)}-\frac{1}{q}\Vert \omega \Vert ^{q}_{q}:\omega\in E,\Vert \omega \Vert ^{q}_{q}=q \mbox{ and } \omega\geq u_{\mu}\biggr\} . $$
(3.13)

Arguments similar to those used in Step 3 and Step 4 show that the above minimization has a solution \(u_{\lambda}\geq u_{\mu}\), which is also a weak solution of problem (1.13). Hence, problem (1.13) admits a weak solution for any \(\lambda\in[0,\lambda^{**})\), This means that \(\lambda^{*}\geq\lambda^{**}\) by the definition of \(\lambda^{*}\). But we already know that \(\lambda^{**}\geq\lambda^{*}\), and therefore \(\lambda^{**}=\lambda^{*}\).

Step 6. Problem ( 1.13 ) admits a nontrivial solution when \(\lambda=\lambda^{*}\) . Let \(\{\lambda_{n}\}\) be a increasing sequence converging to \(\lambda ^{*}\), and \(\{u_{n}\}\) be a sequence of solutions of (1.1) corresponding to \(\lambda_{n}\). By Step 2, \(\{u_{n}\}\) is bounded in X, and without loss of generality we may assume that \(u_{n}\rightharpoonup u\) in X, \(u_{n}\rightarrow u\) in \(L^{q}({\mathbb {R}}^{N})\), and \(u_{n}\rightarrow u^{*}\) a.e. in X. It follows from \(I_{\lambda}(u_{n})v=0\) that, for any \(v\in X\),

$$ \bigl(a+\lambda_{n}\Vert u_{n}\Vert ^{pm} \bigr) \int_{\mathbb{R}^{N}} \bigl(\vert \nabla u_{n}\vert ^{p-2}\nabla u_{n} \nabla v+\vert u_{n}\vert ^{p-2}u_{n}v \bigr) \,dx= \int_{\mathbb{R}^{N}}\vert u_{n}\vert ^{q-2}u_{n}v \,dx. $$
(3.14)

Then, passing to the limit as \(n\to\infty\), we deduce that \(u^{*}\) satisfies \(I_{\lambda}(u^{*})v=0\) when \(\lambda=\lambda^{*}\). Now, it remains to prove that \(u^{*}\) is a nontrivial critical point for \(I_{\lambda^{*}}\). From \(I'_{\lambda}(u_{n})u_{n}=0\) it is easy to deduce that \(\Vert u_{n}\Vert \geq(\lambda_{n} S_{q}^{-q})^{1/(q-p(m+1))}\), which implies that \(u_{n}\) has a lower bound. Next, since \(\lambda_{n}\nearrow\lambda^{*}\) as \(n\to\infty\), it suffices to show that \(\Vert u_{n}-u^{*}\Vert \to 0\) as \(n\to\infty\).

Since \(u_{n}\) and \(u^{*}\) are the solutions of (1.1) corresponding to \(\lambda_{n}\) and \(\lambda^{*}\), we see that

$$ 0= \bigl(I_{\lambda_{n}}'(u_{n})-I_{\lambda^{*}}' \bigl(u^{*} \bigr) \bigr) (u_{n}-u)=X_{n}+Y_{n}-Z_{n}, $$
(3.15)

where

$$\begin{aligned}& X_{n}= \bigl(a+\lambda_{n}\Vert u_{n}\Vert ^{pm} \bigr) \int_{\mathbb{R}^{N}} \bigl(\vert \nabla{u_{n}}\vert ^{p-2}\nabla{u_{n}}- \bigl\vert \nabla{u^{*}} \bigr\vert ^{p-2}\nabla{u^{*}} \bigr)\nabla \bigl(u_{n}-u^{*} \bigr)\,dx \\& \hphantom{X_{n}=} {}+ \bigl(\vert u_{n}\vert ^{p-2}u_{n}- \bigl\vert u^{*} \bigr\vert ^{p-2}u^{*} \bigr) \bigl(u_{n}-u^{*} \bigr)\,dx; \\& Y_{n}= \bigl(\lambda_{n}\Vert u_{n}\Vert ^{pm} - \lambda^{*} \bigl\Vert u^{*} \bigr\Vert ^{pm} \bigr) \int_{\mathbb{R}^{N}} \bigl\vert \nabla{u^{*}} \bigr\vert ^{p-2} \nabla{u^{*}}\nabla \bigl(u_{n}-u^{*} \bigr) \\& \hphantom{Y_{n}=} {}+ \bigl\vert u^{*} \bigr\vert ^{p-2}u^{*} \bigl(u_{n}-u^{*} \bigr)\,dx; \\& Z_{n}= \int_{\mathbb{R}^{N}} \bigl(\vert u_{n}\vert ^{q-2}u_{n}- \bigl\vert u^{*} \bigr\vert ^{q-2}u^{*} \bigr) \bigl(u_{n}-u^{*} \bigr)\,dx. \end{aligned}$$

By the Hölder inequality and compact embedding \(u_{n}\to u\) in \(L^{q}(\mathbb{R}^{N},H)\) we have

$$ \begin{aligned}[b] \vert X_{n}\vert &= \biggl\vert \int_{\mathbb{R}^{N}} \bigl(\vert u_{n}\vert ^{q-2}u_{n}- \bigl\vert u^{*} \bigr\vert ^{q-2}u^{*} \bigr) \bigl(u_{n}-u^{*} \bigr)\,dx \biggr\vert \\ &\leq \int_{\mathbb{R}^{N}} \bigl(\vert u_{n}\vert ^{q-1}+ \bigl\vert u^{*} \bigr\vert ^{q-1} \bigr) \bigl\vert u_{n}-u^{*} \bigr\vert \,dx \\ &\leq C \bigl(\Vert u_{n}\Vert ^{q-1}+ \bigl\Vert u^{*} \bigr\Vert ^{q-1} \bigr) \bigl\Vert u_{n}-u^{*} \bigr\Vert _{q}\to{0} \quad \mbox{as } n\to\infty. \end{aligned} $$
(3.16)

Next, consider the functional \(j:X\to\mathbb{R}\) defined by

$$ j(\omega)= \int_{\mathbb{R}^{N}} \bigl\vert \nabla{u^{*}} \bigr\vert ^{p-2} \nabla{u^{*}}\nabla{\omega}+ \bigl\vert u^{*} \bigr\vert ^{p-2}u^{*}\omega \,dx. $$
(3.17)

Since \(\vert j(\omega)\vert \leq2\Vert u^{*}\Vert ^{p-1}\Vert \omega \Vert \), j is continuous on X. Using \(u_{n}\rightharpoonup{u^{*}}\) and the boundedness of \(u_{n}\) and \(u^{*}\) in X, we have that

$$ \vert Y_{n}\vert \le \bigl(\Vert u_{n} \Vert ^{pm}+ \bigl\Vert u^{*} \bigr\Vert ^{pm} \bigr) \bigl\vert g \bigl(u_{n}-u^{*} \bigr) \bigr\vert \to{0} \quad \mbox{as } n\to \infty. $$
(3.18)

Combining (3.15), (3.16), and (3.18), this forces \(X_{n}\to{0}\) as \(n\to\infty\). Then, using the standard inequality (2.25) in \(\mathbb{R}^{N}\), we have that \(\Vert u_{n}-u^{*}\Vert \to{0}\) as \(n\to\infty\), and thus \(u^{*}\) is a nontrivial weak solution of problem (1.13) corresponding to \(\lambda=\lambda^{*}\). This completes the proof of Theorem 1.2. □